Difference between revisions of "2021 Fall AMC 10B Problems/Problem 13"

(Created page with "==See Also== {{AMC10 box|year=2021 Fall|ab=B|num-a=14|num-b=12}} {{MAA Notice}}")
 
(See Also)
Line 1: Line 1:
 +
==Problem==
 +
IDK Don't ask me.
 +
 +
 +
==Solution==
 +
By similarity, the height is <math>3+\frac{3\cdot}1\cdot2=9</math> and the base is <math>\frac92\cdot1=4.5</math>.
 +
Thus the area is <math>\frac{9\cdot4.5}2=20.25=20\frac14</math>, or <math>\boxed{(\textbf{B})}</math>.
 +
 +
~Hefei417, or 陆畅 Sunny from China
 +
 +
 
==See Also==
 
==See Also==
 
{{AMC10 box|year=2021 Fall|ab=B|num-a=14|num-b=12}}
 
{{AMC10 box|year=2021 Fall|ab=B|num-a=14|num-b=12}}
 
{{MAA Notice}}
 
{{MAA Notice}}

Revision as of 10:04, 23 November 2021

Problem

IDK Don't ask me.


Solution

By similarity, the height is $3+\frac{3\cdot}1\cdot2=9$ and the base is $\frac92\cdot1=4.5$. Thus the area is $\frac{9\cdot4.5}2=20.25=20\frac14$, or $\boxed{(\textbf{B})}$.

~Hefei417, or 陆畅 Sunny from China


See Also

2021 Fall AMC 10B (ProblemsAnswer KeyResources)
Preceded by
Problem 12
Followed by
Problem 14
1 2 3 4 5 6 7 8 9 10 11 12 13 14 15 16 17 18 19 20 21 22 23 24 25
All AMC 10 Problems and Solutions

The problems on this page are copyrighted by the Mathematical Association of America's American Mathematics Competitions. AMC logo.png